2012 AMC 12B Problems/Problem 24
Contents
Problem
Define the function on the positive integers by setting and if is the prime factorization of , then For every , let . For how many s in the range is the sequence unbounded?
Note: A sequence of positive numbers is unbounded if for every integer , there is a member of the sequence greater than .
Solution
Solution 1
First of all, notice that for any odd prime , the largest prime that divides is no larger than , therefore eventually the factorization of does not contain any prime larger than . Also, note that , when it stays the same but when it grows indefinitely. Therefore any number that is divisible by or any number such that is divisible by makes the sequence unbounded. There are multiples of within . also works: .
Now let's look at the other cases. Any first power of prime in a prime factorization will not contribute the unboundedness because . At least a square of prime is to contribute. So we test primes that are less than :
works, therefore any number that are divisible by works: there are of them.
could also work if satisfies , but .
does not work.
works. There are no other multiples of within .
could also work if , but already.
For number that are only divisible by , they don't work because none of these primes are such that could be a multiple of nor a multiple of .
In conclusion, there are number of 's ... .
Solution 2
Say a number is if the sequence is bounded; otherwise is .
Notice that for all ; an inductive argument leads to the conclusion that multiples of an interesting number are interesting.
We have ; since iff , we have is the least power of that is interesting. There are multiples of that are .
We have ; since iff , we have is the least power of that is interesting. There are multiples of that are .
We have , so a number of this form is interesting iff or ; they are already counted above.
Integer is interesting (resp. boring) iff is. A square-free integer is boring, since .
Let be the subset of consisting of integers whose prime factorization only contains exponents 2 or higher. It suffices to check : all interesting numbers in are multiples of interesting numbers in .
Let's first look at all the prime powers (these have no other multiples in ). They are: , , , and . Only is interesting but it has no other multiples (even in ).
We are left with such that either or .
If then is , or or and all are boring. If then , or or (all three are boring) or , which is interesting, but, of course, has no other multiples.
We have multiples of , multiples of , , and for a total of ... .
Video Solution by Richard Rusczyk
https://artofproblemsolving.com/videos/amc/2012amc12b/276
~dolphin7
See Also
2012 AMC 12B (Problems • Answer Key • Resources) | |
Preceded by Problem 23 |
Followed by Problem 25 |
1 • 2 • 3 • 4 • 5 • 6 • 7 • 8 • 9 • 10 • 11 • 12 • 13 • 14 • 15 • 16 • 17 • 18 • 19 • 20 • 21 • 22 • 23 • 24 • 25 | |
All AMC 12 Problems and Solutions |
The problems on this page are copyrighted by the Mathematical Association of America's American Mathematics Competitions.